Nmss.org.np

1.name five causes of distended uterus. 11.what is the active management of third stage of labor? i.v. ergometrine is given following the birth of anterior shoulder. the placenta is delivered by the controlled cord traction soon after the endometrial polyp delivery of the baby availing first uterine contraction. if the first attempt fails, another attempt is made after 2-3 minutes failing which another attempt is made at 10 minutes. 2.give the composition of ors. if this still fails, manual removal is done. nacl 3.5 g (60 nm) finally oxytocin 5-1- units i.v. is given. kcl 1.5 g (20 nm) sodium citrate 2.9 g (30 nm) 12.give the clinical features of iridocyclitis. glucose 20 g (110 nm) circumcorneal injection occurs. there may be white specks visible on the posterior surface of the 3.what are the risk factors of surgical infection? cornea. (keratitic precipitates) · poor surgical technique the pupil may be constricted and irregular due to the formation of · metabolic disease (diabetes, jaundice) adhesions (posterior synechiae) between the edge of the pupil and the · immuno suppression anterior surface of the lens. · poor perfusion (local ischemia or systemic shock) · foreign body material 4.name the causative agent of: short answer questions: i. plague yersinia pestis five complications of radiotherapy? ii. anthrax bacillus anthracis five complications of dka (diabetic ketoacedosis)? iii. diphtheria corynebacterium diptheriae extended program of immunization iv. trachoma chlamydia trachomatis five clinical features of intussusception? v. leprosy mycobacterium leprae treatment of csom (chronic suppurative otitis media) treatment of ectopic pregnancy. 5.what are the complications of ludwig's angina? usg in third trimester. assessment of pem generalised infection classification of leprosy five skin manifestations of hiv aids methods of nutritional assessments? 6.what are the possible adverse reactions of blood transfusions? · congestive heart failure multiple choice questioins: · transfusion reaction a newborn child with heart rate 88, poor cry, minimal respiratory effort,hypotonic with slight movements of the arm . what is the apgar · coagulation failure · thrombophlebitis a child with pustules along with maculopapular rashes. what is the diagnosis? ans: chicken pox 7.name the complications of pneumonia. para-pneumonic effusion least antiemetic inducing preanesthetic drug: thiopentone, propofol, na valproate, ketamine ans. propofol retention of sputum causing lobar collapse development of thrombo embolic disease pneumothorax – particularly with staphylococcus aureus rx of three spike wave (absence seizure)?ethosuximide, na valproate, 8.give five causes of pathological sinus bradycardia. rta with head injury with gcs 9 most likely to be given fluid is? rl, ns myocardial infarction ,5% dextrose, dns 7m child with b/l wheeze during winter ans bronchilolitis sinus node disease (sick sinus syndrome) cholestatic jaundice a dyspnic pt. comes with decreased vical freitus, dull on percussion, 9.name the different cerebellar signs. trachea shifted to opposite side ans. pleural effusion postural changes and alteration of gait thyroid surgery – which nerve get damaged, ans. recurrent laryngeal disturbance of voluntary movement (ataxia) dysdiadochokinesia disturbances of reflexes (pendular knee jerk) appendix perforate which one is true: a. not common in <2yrs, disturbance of ocular movement (nystagmus) management is same in pregnanat, common in children, disorder of speech (dysarthria) murmur – ans: rheumatic heart disease 10.name five anti retro viral drugs. indication for caeserian section fb in ear extraction – what is the temperature of the warm water 31- 32, 33-34, 35-36, 36-37 pre vaccination requirement an 18 year old girl brought to er with respiratory depression, miosis, • idiopathic hypertrophic subaortic stenosis profuse secretions, bradycardia etc. ans: op poisoning in which bone disease methotrexate used: paget's, perthe's, oa, 8. where do you find mid diastolic murmur, pansystolic murmur, and early diastolic murmur altogether? how long scabies remains without treatment, 3wk, 3 month, 1yr, please search the answer yourself. 9. a patient with a fracture of neck of femur comes with a complaint all are present in rigor mortis except: mol. death, duration of 12-18 hr, of dyspnoea. why do you think has happened? (2 more option could be remember) what is the least common of eclampsia : headache, proteinurea, blurring of vision, epigastric pain 10. which of the following fractures is best healed? what is a felon? ans: a suppurative abscess in distal phalanx of finger. 11. painless hematuria occurs in: why there is hypertension in spinal anaesthesia ans is vasomotor please find the answer yourself. 12. carcinoma of bladder is a type of: _ dissociation is more common in mania, schizophrenia, depression, what do you give in alcohol withdrawal? thiamine, haloperidole, vit. 13. a 70 year old man comes with a history of dysuria, what may be what do you give in vt ans is xylocard. the most liable cause? a 3 yr child weighing 10 kg with sever dehydration can't take orally, to answer this question be familiar with fluid calculation. 14. which is the most commonly injured organ during penetrating injury to abdomen? 6 wk pregnant, come with vaginal bleeding, usg suspect ectopic pregnancy which one is the best next possible investigation, beta hcg, leproscopy, x-ray abdomen 15. rashes and abdominal pain is suggestive of – henochs scholein purpura. a man dies on plain crash, which one is identifying feature: skull x- ray, pelvic x-ray, dental x-ray, chest x-ray. 16. which of the following is the diagnostic investigation of choice in case of trauma to chest? chemoprophylaxis is indicated in all except, pellagra, hepatitis a, 17. which of the following is the drug that causes gum hypertrophy? causative agents for portal hypertension ans. cistosomiasis 18. which of the following causes joint pain as the most common where is the defect in muscular dystrophy? ans. dystrophin a pt. come with chronic non healing ulcer suger is 26 mmole what may be the possible diagnosis - tropical ulcer, tubercular ulcer, 19. which of the following drugs causes steven johnson syndrome? malignant ulcer, syphilitic ulcer which organism causes anaemia - scaris lubricoud, t. saginata, t. 20. what is the indication for tympanicectomy? trichuriasis, pig tape worm 1. what is called the poor mans meat? 21. colustrum contains the highest amount of- 22. allergic conjunctivitis- 2. which of the following is the commonest site of endometriosis? 23. what is the clinical finding in whooping cough? subconjunctival hematoma 3. what is the target value of imr? 24. what is the clinical symptom that suggests psychosis? 4. what is the dose of tt given in pregnancy? in case of previously un-immunized cases, two doses of 0.5 ml tt im at 25. neurosis – transient 6 weeks interval, first between 16-24 weeks. in previously immunized, a booster dose of 0.5 ml im in last trimester. 26. which of the congenital heart disease doesn't cause heart failure? tetralogy of fallot 5. in the health centre a boy 10 years old comes with colles fracture. which kind of anaesthesia would you give? 27. what is the genetic make up in a case of turner's syndrome? infiltrating anesthesia 28. white discharge is seen in cervicitis. ( there were other options as 6. what would be the most suitable treatment for a fracture of neck of femur in a 40 year old male who comes to you after two days? closed reduction and internal fixation 29. commonest agent that cause vaginitis- candida albicans 7. where do you find pulsus biferans? 30. which of the following is not the clinical feature of fibroids – • combined aortic stenosis and regurgitation • pure aortic regurgitation 31. which of the following is the most dangerous tumour of the female was there on the follwoing day. so this is our small effort to share this genital tract? – ovarian tumour questions that we came in our nmle exam. we tried our level best to recollect all the questtion but we couldn't you know its human mind. 33. which of the following is the commonest causative organism of upper uti? – e coli so, our suggestion is "healthy internship is the best way to pass nmle 34. post streptococcal glomerulonephritis causes nephrotic syndrome 35. when is iucd applied? – 5th day of menstruation 36. which nerve is damaged in carpel tunnel syndrome? –median bes of luck for those who will appear in the future nmles. 37. which of the following viruses is a dna virus? –hepatitis b 38. vertical transmission is seen in – hepatitis b  give five causes of gradual painless loss of vision. 39. hepatocellular carcinoma can be caused by- hepatitis b virus 40 yr old man present with engorge vein in the neck 40. what is the concentration of oxygen in umbobag? ans is 20.009 diagnosed to have retrosternal goiter, write four other symptoms associated with this disease.? 41. what would be the most suitable treatment for a boy with  name five regional organizaion of who with their location. hypothyroidism? l-thyroxine  south east asia – new delhi, india. 42. what is the most common cause of post partum hemorrhage? – brazzavilli, congo. – washington dc, usa. 43. which of the following is the feature of beurgers disease? absent – copenhgen, denmark. – alexandria, egypt. – manila, philipins. 44. allergic dermatitis is caused by- nickel 45. which of the following is used as a protection against x-ray  give five cause of postmenopausal bleeding. radiation? - lead  give five causes of hoarseness of voice.  give five causes of cyanotic heart disease. 46. what do these features suggest: -acute viral conjunctivitis  name five organism responsible causing dysentery in children in nepal. 47. trachoma is caused by chlamydia trachomatis  a 50 yr old man smoker and alcoholic since last 30 yrs presented with icterus and abdominal distension what is the 48. in children what is the most common cause of unilateral nasal most likely diagnosis? name three investigation to diagnose obstruction with foul smell? foreign body obstruction. the condition. (ans: alcoholic liver cirrhosis. inv – lft, ascitic fluid analysis, ugi, usg ect) 49. what would you do in a hypertensive patient who comes with epistaxis? anterior nasal packing and glycerine. a 14 yr old child gives the history of continuous fever since last seven days. on examination temp 102 f, pulse 80/min, 50. all are true about mumps except- bp 120/80 mm of hg, chest and cvs normal, spleenic tip • always bilateral palpable. what is the most likely diagnosis? write two • associated with testicular pain investigations to support your diagnosis and treatment plan. • vaccine is effective  8 week pregnant woman presented with pain abdomen and brownish discharge per vagina give your differential 51. what is the commonest mode of infection of leprosy? – droplet diagnosis and how will your diagnosis.  name five components of phc (primary health care) in nepal. 52. partial dislocation of joint is called- sublaxation. 53. which of the following is secondary skin lesion? short questions: 1. 60 yr old man presented with generalized swelling of the body and shortness of breath. on examination there is pulsatile liver. what is the expected finding? a. pulsus bisferines 54. three month infant with projectile non-bilious vomiting and b. prominent v wave on jvp palpable mass in the right hypochondrium c. systolic murmur increased during expiration • duodenal atresia d. oxygen saturation • congenital hpertrophic pyloric stenosis • gastric cancer 2. a 14 year old boy presented with fever, chest pain, cough on nmle question september 2, 2006 - 12-09-2006, 07:42 pm examination temp 101 f, percussion – dullness in the right subscapular region with bronchial breath sound. the day before exam was a horrifying situation, receiving different a. bronchipneumonia is the diagnosis calls .and question papers. however after receiving question paper we b. wheezes are expected had a mixed feeling, neither happy nor sad!!! none of the questioned c. crackles are expected a. occurs in nulliparous d. all of the above b. aggravted by cough c. pessary is the permanent treatmaent 3. a 60 yr old man presented with left sided chest pain, on ecg there is st segment elevation in lead ii, iii and avf with reciprocal depression 14.nephrotoxic drug is in lead avl and i. what is the diagnosis? a. anterior wall mi b. posterior wall mi c. cephalosporin c. inferior wall mi d. lateral wall mi 15.drug that is avoided in pregnancy 4. 16 yr old boy gives the history of continuous fever of last 7 days on examination temp 104 f, pulse 90/min with coated tounge, chest, cvs, p/a are normal, what is the probable diagnosis? 16.commonest complication of ovarian cyst- c. enteric fever 5. 12year old male c/o of fleeting joimt pain and arthritis with subcutaneous nodule .0n ecg there is increased pr interval .what is 17. a boy c/o fever ,headache and vomiting showed maculopapular rashes all over the body . the organism is b.rheumatic fever b. meningococcal 6. 60 yr old female c/o swelling in the body ,cold intolerence, 18. a 3yr old boy is c/o cough since the last 100dys with repeated slowness in activity. o/e pulse -54/min with delayed relaxation of knee episodes of vomiting. the diagnosis is jerk. her blood examination will show- a. whooping cough a.high cholesterol c. brondhiolitis 19. 70 yr old man with dm2 developed bpof 160/100 .the drug of 7. serum cretinine level in mg% is 20. imr is mortality of 8. following antitb drug is b. <30 days old d. <10 yr old 21. xerophthalmia is caused due to deficiency of 9. 8+weeks old pregnant presented with pain abdomen and vaginal vitals stable.diagnosis is a. incomlete abortion b. complete abortion c. threatened abortion 22. a 72 yr old man is c/o gradual painless of vision with cloudy d. ectopic pregnancy papillary reflex .the diagnosis is 10. a doctor is graded good in the society b. retinal detachment a. by the knowledge he posseses b by the skill he possesses c.good behaviour in the society 23. quinsy is pus in a. peritonsillar space 11. socialization means b. retropharyngeal space a. to adapt social norms and values c. submandibular space b. started in soviet union c. socialist revolution 24. frontal sinus in x-ray appears in 12 contraceptive preventing std and hiv 25. a 36 yr old man presented on and off fever ,weight loss and abdominal distention.on ascetic tap tc-300/cu.mm dc - 13 true about .uv prolapse 98%lymphocytes protein-4gm%.the diagnosis is b.age estimation c.rape victim case 37.gcs is the estimation of 26. 25 yr old man is c/o fever on and off ,weight loss and sob. o/e b.<2 incompatible with life decreased breath sound on rt.side with dull percussion on rt.subcoastal c.was introduced by dr.ferguss glasgow region .best inv. would be a.pancreatitits is a known complication c. pleural tappingand biochem analysis b.orchitis occurs in <1 yr old c.vit aa def can occur 27. insulin is secreted by 39.2yr old male is c/o red congestion of eye, fever with maculopapular rashes .the diagnosis is b. conjunctivitis 28. a 64 yr old man is having icterus with distended gall bladder 40.neck of indirect hernia is in a .ca pancreas-head a.lat to inf .epigastric artery b ca pancreas tail b.medial to inf epigastric artery c.superficial to deep ring d.deep to inguinal ligament 29. a patient on 3rd post op day developed low urinary output - 41.16 yr old male with pain in the left testis sudden in onset.rt. testes 10ml/hr .cvp-2cm bp 90/60. what will you do? diagnosis is – c. fluid challenge 30. 7th day old neonate with features of septicaemia .the drug of 42.swelling in post. triangle of neck in 3yr old child with transillumination a. penicillin +gentmycin positive cyst is . b. iv ceftrixone c. penicillin only c.cystic hygroma 31. 60yr old nam c/o ill health ,wt loss .on inv tc-48000 dc-97% lymphocytes with predominant small ones without spleenomegaly. the 43.colles fracture occurs in diagnosis is – 44. green stick fracture occurs in 32. hepatitis causing epidemic is 45.hormone that can be given orally 33. fat droplets in faeces is seen in a. chronic pancreatitis b. acute panretitis 46.the variable whichdivides the distribution into two equal parts 34. 1 yr old with fever, cough and sob in the winter season. the diagnosis .the diagnosis is a. acute bronciolitis 47.clinical staging is done in 35. development of second molar tooth occurs at the age of c.ca. endometrium 48.6mth yr old presented with meningitis the common organism is a.streptococus pneumoniae 36. which of the following require consent? c.grp b streptococcus 49.alcohol withdrawl causes c.conversion disorder 62.!st sign in diabetic retinopathy 50.hysteria is a type of c.retinal haemorrhage c.psychosomatic disorder 63.a child with acute severe asthma should be immediately treated 51.most common cause of rupture spleen b.nebulised salbutamol d.infectious mononucleosis 64.absolute indication for amputation of foot 52.female sex worker has a highest risk of transmitting hiv whenthere 65.there is increasing size of toxic nodule which is excised.cause of c.urethral discharge a.suspicion of malignancy d. molluscum contagiosum b.cosmetic purpose 53.health for all was declared at 66.steroid is indicated in 67.bence jones protein is seen in 54.amother with hiv should avoid a.multiple myeloma a.breast feeding 68.argyl robertson pupil is seen in 55.64 yr old man presented with haematuria with no history pain abdomen and feverand burning micturition. b.retinal detachment 69.time of iucd insertion is a. after menstruation 56.common cause of rewctal bleeding in a1yr old child is c 1st day if menstruation 70.cast in both bone fracture should be applied from a. forearm to wrist 57.30 yrs old man presented with lump in the breast,firm innature b. below elbow to neck of metacarpal with no palpablelymph nodes.the diagnosis is c above elbow to neck of meta carpal 71.a patient with cervical injury is is brought to er .ist thing to do is a.airway maintainance 58.8th post op day a man developerd sob with no other chest findings and swelling of left leg .the diagnosis is a. chest infection 72.2nd stage of labour occurs after b.pulmonary embolism a.delivery of baby b .full dilatation of cervix c.onset of labour pain 59.true about corpus luteum a.secretes mainly progesterone 73.true about brachial plexus b.lasts for 21 days a.arises from c5-t1 c.made up of theca cells border of arrangement is vein artery and nerve 60.a dead body was found warm and flaccid.the death has occurred in 74.caloric test is done for the function of 61.haemphiliaa is due to deficiency of 75.a6month yr old female ishas sob , previous6 cynotic attacks and has clubbimg.the had 2 lit urine output, 1.5 litre ng drain, abdominal distension, no flatus, absent bowel sound. on inv. potassium 2.4 mmol/l , sodium : a.tetralogy of fallot 135 mmol/l, urea : 7, creatinine : 75, tc : 10,000 /cu.mm n 73 %.l 17 %, fbs : 5 mmol/l a) what is the clinical condition called b) what is the abnormal lab. finding 76.a new yr old newborn has short stature with webbed neck ,slanting c) what could be the cause eyes,curved nail .the diagnosis is d) management of this condition a.turner's syndrome b.kliene felter's syndrome dear friends, we had promised you all to post here the questions of the 7 c.mucopolysaccharides nmle held on 4th february 2006, we tried our best and have collected as many questions as we could and here they are. the examinees who 77.which of the following is a pre malignant condition ? we met at the exam place felt that the examination was getting more and more standard and obviously more difficult these days. i think it's a good move. and as far as these questions are concerned, i think it would be better if you search the answers yourself. so, i have not listed the answers here and about some of the mcqs i could not collect the exact options, and i have only mentioned the questions so please, - compiled by: dr. saurav & dr. khem, kmc help yourself…….i am extremely sorry about that. i wish this will be of some help to you all…. nmle december 23, 2006 short questions: held in iom nursing campus, 1. list five causes of metabolic alkalosis. time 8.00 - 11.00 am 2. list the complications of colostomy. 3. give the incubation periods of: 1) 35 yrs old chronic alcoholic man taking alcohol since last 10 yrs has bp 200/120 mm hg. a) what are the possible organ damage due to his condition ? b) what would you advise him for investigation ? c) what suggestion would you give to him. ? 4. give the full forms of: 2) 35 yrs male presented with pain epigastrium and malena. on ugi endoscopy pt has du. a) what is the causative organism ? b) what t/t would you prescribe him 3) contraindication of ocp 5. a 60 year man comes with a mass in upper abdomen. the usg 4) complication of frontal sinusitis showed a cystic mass, what could be the differential diagnosis? 5) causes of postmenopausal vaginal bleeding in 60 yrs female 6. outline the basic management and complications of ovarian cyst. 6) write the health determinants of nepal 7. give the indications of following drugs: 7) how will you remove hardness of water 8) case question : d/d and diagnostic criteria for rheumatodi arthritis iii. streptokinase iv. sodium valproate 9) 14 yrs of male presented with vomiting, pain over lt testis since 4 v. d- penicillamine hrs. the patient looks tense and toxic a) what is your clinical diagnosis 8. what are the possible routes of spread of liver abscess? b) what investigation will u do to support your diagnosis c) what is the management? 9. what are the criteria for tracheostomy? d) what will u advice for prophylaxis to avoid such in future. 10) jaundice in neonate developed in 4th day ( case ) 10. a child, 2 years old, comes with a complaint of stridor and fever. a) what clinical sign will u look for what could be the possible differential diagnosis? b) investigation 11. a 30 year old female comes with a swollen right eye. what could 11) 35 yrs old male farmer presented with pain, red eye, congestion, be the possible causes? photophobia over lt eye. pt also gives h/o paddy plant trauma in the field. on exam pt had ant hypophyon 3x 2 mm size,. a) what are your d/d 12. name four absolute contraindications for oral contraceptive pills. b) what is your clinical dx c) what will you examine and investigate d) write 4 drugs used in t/t 12) intestinal bowel resection done about 1.5 m, on 5th post op day, pt 1. most of the thyroid cancers arise from: i. parafollicular cell ii. lymphoid follicle iii. follicular cell iv. connective tissue 20. retrograde ejaculation is common after …………… 2. which of the following anti-hypertensive drug is contraindicated in 21. in pem there is: i. weight for age is decreased ii. height for age is decreased iii. weight for height is decreased iii. hydralazine iv. weight is increased but height is decreased iv. ace inhibitors 22. treatment of choice in stage iib hodgkins disease is 3. anti-hypertensive of choice in pregnancy induced hypertension is: ii. radiotherapy iii. physiotherapy iii. calcium channel blockers iv. surgical excision iv. vasodilators 23. drug used for pediculosis capatis in children is …………… 4. an infant presents with persistent jaundice, what could be the most 24. flasked shaped ulcer is found in i. bacillary dysentery ii. amoebic dysentery i. most commonly occurring value ii. the middle value iv. ulcerative colitis iii. least occurring value iv. the highest value 25. thrombocytopenia is not a feature of 6. the probability of a test to come as positive in diseased person is iv. all of the above iii. positive predictive value 26. antithyroid drug of choice in elderly patient is ………………… iv. negative predictive value 27. in quartan malaria, fever spikes over every 7. a child suddenly presents with dyspnoea, the most common cause ii. foreign body aspiration iii. acute excacerbation of copd here are few questions asked in nmle 4th feb,2006 iv. pulmonary embolism 8. downs phenomenon is: 1.write down the full form of the followings 9. ovarian artery is a branch of a) unicef=united nations international children's emergency fund 10. dangerous placenta previa is …………. b)sars =severe acute respiratory syndrome c)mmr =measles mumps rubella 11. tb becomes non-infectious by ……………. d)phc =primary health centre 2.write down the incubation periods 12. thyroglossal cyst is a cyst of …………………. a) measles=7-14days(n.r.10days) 13. after anesthesia, failure to breathe is due to …………. b) mumps =12-21days("18days) c)hepatitis b=6weeks-6months(12weeks) d)typhoid fever=7-21days 14. fracture of neck of femur in 70 years elderly can be best treated e)chicken pox =14-21 days(21days) 15. treatment duration in lepromatous leprosy is ……………. bacillary dysentry (2-5days) amoebiasis14days -months(21days) 16. oligomenorrhoea is ………………. malaria 8days -months poliomyelitis 3dyas -21days hepatitis a 2-6weeks 17. mycoses fungoids is ………………. leprosy =years(2-5years) rabies =variable (2-8weeks) 18. infertility is said to be present when the couple can't conceive by ……………… 3.sources of infection(pyogenic liver abscess) -infection through portal vein =acute appendicitis ,acute amoebic 19. a newly married couple wants to delay pregnancy for next 2 years so what will be the best contraceptive of choice for them? -infection through the common bile duct =stricture of the cbd,cholangitis etc. -infection through the hepatic artery =septicaemia and pyaemia etc. -infertility is treated with clomiphene .80%ovulate and 40%conceive -extension abscess=penetrating injuries etc. -infection through umbilicus =neonatal umbilical sepsis giving rise to treatment of benign ovarian tumour in young patient 4.drug of choice -ovarian cystectomy methotrexate=rheumatoid arthritis -ovariotomy (or salpingo -oophorectomy) phenytoin=epilepsy when any doubt aboutn innocence of the tumour streptokinase=myocardial infarction parous women and age >45 years -total hysterectomy with bilateral salpingoophorectomy 5.55 years old man underwent colostomy , now you would like to ovarian tumour with pregnancy explain him about the complication ,hat are the complications? -continue pregnancy upto full term if no complication -if no delivery occurs remove the tumour in puerperium -colostomy bleeding ,necrosis,retraction,prolapse,colostomy -if tumour impact in the pouch of douglas caeserean section and remove the tumour in same sitting -if causing obstructed labour.caserean section and remove tumour 6.67 years old man on usg mass on the upper abdomen diagnosis was -if any complication immediate surgery not made what are your provisional diagnosis? -amoebic hepatitis treatement of malignant ovarian tumour: -ca of the liver -subphrenic abscess -intussusception definitive surgery :total hysterectomy with bilateral -pseudopancreatic cyst salpingoohorectomy ,appendicectomy and omentectomy(partial /total) 7. swelling over the right eye ,what are the possible diagnosis? conservative surgery:unilateral salpingoophorectomy (young patient -corneal oedema(glaucoma) ,desire offspring ,tumour limited only in ovary ) -hordeolum internum,acute dacryocystitis -orbital cellulitis -orbital abscess -endophthalmitis 8.45 years female,usg diagnosed ovarian cyst .definitive management adriamycin 50mg/m2 what complications could occur if not managed in time? cyclophosphamide 500mg/m2 cisplatin 50mg/m2 -non neoplatic enlargement of ovary cp=cyclophosphamide 750-1000mg/m2 a)follicular cyst -disappear spontoneaously within a few weeks to cicplatine 50-100mg/m2 months when symptoms like amenorrhoea are prolonged stimulation of post-ovulatory change by administrating oral medroxy progestrone drugs are given i.v.on day 1 every 4 weeks interval for 6-8 cycles. 10mg tds over a period of 5-7days .clomiphene citrate 50mg given orally for 5 consecutive days helps to induce ovulation and brings about menstuation or pregnancy b)follicular haematoma:asymptomatic/no clinical significant c)lutein cysts of the ovary : granulosa lutein cysts most cysta resolve in due course of torsion of the pedicle time,observation intracystic haemorrhage theca lutein cyst=spontaneous regress d)pcos(polycystic ovarian syndrome) rupture of a cyst usg -subcapsular cysts of varying size intestinal obstruction treatment -wt. loss help in restoring the hormonal milieu to some 9.write down the criteria for tracheostomy respiratory obstruction -avoid cigarette smoking infection -acute laryngo-tracheo-bronchitis -oestrogen supresses androgen and adrenal production acute epiglottitis,ludig angina -dexamethasone 0.5mg /prednisone 5mg at bed time decreases trauma - injury of larynx and trachea androgen production neoplasm - benign and malignant neoplasmof larynx ,pharynx,foreign -hirsutism is treated with cyproterone acetate oedema of larynx - tubercular meningitis retained secretion -cva ,head injuries etc. - pyogenic meningitis paingul cough -chest injuries ,multiple riv fractures, pneumonia - febrile convulsions ,aspiration of pharyngeal secretion 3. cadaveric spasm indicates. respiratiory insufficiency - patient was in motion b4 death - starts in voluntary muscles 10.causes of metabolic alkalosis(high ph,high co2) - similar to rigor mortis -due to loss of acid eg.vomiting nasogastric suction  who ors contains -hco3 may be retained in exchange for loss of cl- as in diarrhoea -dut to excessive administration of alkali eg. nahco3 administration antacid abuse -when h+ are lost in excess in exchange for k+eg.severe - glucose in excessive amount -primary or secondary hyperaldosteronism. - dextrose in less amount 11.2 years child brought to you with a h/o fever and stridor what are  ringer's lactate your possible diagnosis -acute laryngitis ,laryngomalacia - contains more na+ than in plasma -acute epiglottitis - is used because it contains all electrolytes in equilibrium with -laryngeal trauma,laryngeal oedema - contains less na+ than in plasma - contain less k+ in plasma  right recurrent laryngeal nerve hooks around 12.contradication of oral contraceptive pills - right main bronchus - subclavian artery circulatory disease - arterial /venous thrombosis severe htn  60 years old patient bph with htn is under going for turp valvular heart disease ,ihd focal migraine liver disease anaesthesia which drug will you give? others-pregnancy undiagnosed genital tract bleeding 8. g2p1 32 weeks pregnant came with vaginal bleeding & painless contractions, what is diagnosis? -age>35 years - preterm labour -bronchial asthma - premature labour - abruption placenta i tried to gather all of the questions ,but some are missing if u can try 9. g2p1 37 weeks pregnancy came but fhs negative & she was having to grab those , i did my labour best to answer as well do comment me or else try to search the best answers good bye pain, what will you do? in near future i will provide other keep waitin bye bye - wait for house officer multiple choice questions 10. nifedipine causes  a patient came with swelling in inguinal region which increased in size,tachypnoea,tachycardia,absent bowel sounds & in painful condition. - increase cardiac output - decreases cardiac output - strangulated hernia 11. child was having <70% of body wt & later he developed - obstructed hernia oederna,15 days fever, what is likely diagnosis? - irreduciable hernia 2. 10 month old child fever & convulsions. tlc=250 dc = 90% - marasmic kwashiorkor lymphocytes=10% protein=26% 12. definite host for e. granulosus is - khaki coloured skin 25. a mother brought 18 month old child having fever 104degree fever 13. most common cause of otitis media is children protein ,glucose is normal in c.s.f what is likely diagnosis? - ossicle fusion - febrile convulsion 14. second stage of labour in primigravida lasts for 26. cardiac output in pregnancy isn't affected by- - heat surrounding - ---------------- 15. ectopic pregnancy mostly occurs at 27. which of the following isn't intracranial complication of asom? - facial nerve paralysis - lateral sinus thrombophlebitis - vaginal moniliasis 28. infant mortality rate in 2006 is - endometrial cancer 17. most common type of breast cancer in nepal. 29. a child (malnourished) bought by her mother to health camp has what is diagnosis? - toxocara endophthalmitis 18. 35 yrs old lady came with painless thyroid nodule what will you 30. primigravida excessive vomiting in pregnancy which has - antithyroid drugs deleterious effect upon health of mother & fetus, what is it? 19. e. granulosus is transmitted by ingestion - of faeces, containing ova, of dog - of faeces , containing ova , of sheep - hyper emesis gravidarum - of liver of sheep containing e. granulosus 31. diabetes mellitus is best diagnosed by- 20. direct questions are allowed in - p.p blood sugar > 11.1mmols - cross examination - fasting blood sugar > 7mmols/l - dying deposition - dying declaration 32. putrefaction is delayed in which poisoning? 21. which of following doesn't cause liver cirrhosis? - alpha 1 antitoxin deficiency - cystic fibrosis - haemochromatosis 33. which is false about caput succedaneum? - appears immediately after birth 22. most suitable outcome of tetanus occurs when - disappears at 2-6 weeks - patient comes late in hospital - patient has short course of disease 34. which disease is transmitted both by water & air? - patient has long course of disease 23. nutritional deficiency is best corrected - at nutritional training centres 35. which doesn't transfers active immunity? - antitetanus toxin - when person takes enough food (available) - cholera vaccine - rabies vaccine 24. which of the following is less likely in gas gangrene? 36. cvp reflects - rt atrial pressure 48. diabetic patient who is hypertensive came & on fundoscopic - cardiac output examination flame - cerebral & renal perfusion shaped haemorrhage seen. what is next thing you would like to do? 37. patient with ascitis, when per cussed an abdomen it was resonant as the percussion is done lat side it was dull. patient is turned & again lateral side s per 49. continuous murmur in children it's resonant? what is sign called? - aortic stenosis - shifting dullness 50. 2 yrs old child cyanosis in lip & clubbing in fingers. what is 38. lady came with bilateral nasal obstruction, poor hygienic condition 51. which of following is fluid filled lesion? from nose noticed by others. what is diagnosis? - hyper tropic rhinitis - rhinosporidiosis 52. 50 yrs old patient has itching & on scraping silvery white lesions 39. antidote for warfarin is are present. what - protamine sulphate - dermatophytosis 40. person came with abdomen pain, tachycardia, liver dullness 53. which of following contains rash of all stages? - acute pancreatitis - acute appendicitis - perforated peptic ulcer 54. a school teacher has feeling that every time she has to beat 41. drug which is my dricetic with little cycloplegic action. children & she has guilty for that. she tried a lot to avoid this thinking from her mind. 42. in rickets which of the following is seen 55. thought block is seen in - alkaline phosphates 43. causes of death in crf 56. anti maniac drug is 44. haemodynamic complication of massive blood transfusion. - citrate intoxication 57. patient has sputum +ve tuberculosis & x-ray showed cavitary - thromboembolism lesion which drug would you like to give? 45. which drug of choice in op poisoning? 58. baby has cough with subconjunctival haemorrhage - whopping cough 46. pseudomonas is treated with 59. the chance of getting true positive in case is called as 47. common cause of corneal ulcer in nepal 60. the patient had small cell carcinoma of lung, what is treatment that would like to give? - who are 1year old 72. most common problem of lbw but appropriate for date babies in 61. the patient had small nodule in breast which undergo change - fibro adenosis 73. hepatitis serum conversion is best denoted by 62. during subtotal thyroidectomy patient developed sudden on set of - anti – hbsag what is likely cause? - tracheal haematoma 74. the child has abdomen pain, non bilious vomiting, what is - tracheal perforation - congenital hyper tropic pyloric stenosis 63. structures not to be preserved in modified radical mastectomy is - intestinal obstruction - pectoralis minor - long thoracic nerve 75. 50 yrs old female came with complaint of vaginal bleeding. what - thracodorsal nerve 64. cubitus varus deformity is seen in - supracondylar fracture of human - lateral condylar fracture of human - cervical biopsy - galaezzi fracture - endometrial biopsy 65. most common complication in intracapsular fracture of neck of 76. boy has sudden onset of organizing pain with vomiting on left worsened when testes was elevated. what is diagnosis? - testicular torsion - epididymo – orchitis 66. foot drop because of trauma is popliteal region is seen in 77. ejaculation is controlled by - common peroneal nerve palsy - sciatic nerve palsy - tibial nerve palsy - parasympathetic 67. maturity of fetus in usg is best determined by 78. g3p1 on 36wks pregnancy painful urine contractions. what is - bitemporal diameter 68. hypertensive male has right sided hemiplegia, which of following - preterm labour 79. 2 yrs old girl had webbed neck, co-arctation of aorta. what is - blowing of cheek in & out with respiration - tongue pointing towards side of lesion - turner's syndrome - down's syndrome 69. on auscultation of right side of adult male at interscapular, infra - klinefelter's syndrome inframammiliary regions following are seen – 80. at high altitude person developed acute mountain sickness. what is a. absent breath sounds c. decreased chest movement. what is diagnosis? - bring him down - rt. lower lobe lung collapse - rt. medial lobe lung collapse - pleural effusion - pericardial effusion 81. l.d. bodies're found in 70. neonates are those - who are <28 days old - who are more than 28 days old - who are 1 month old - who are after 28 weeks pog & 7 days of life 82. g2p1 on labour for 6hrs, membrane ruptured, head station +2, 71. perinates are those malrotated what is - who are late fetuses but early neonates 1. which of the following is the most common hernia in women? a.femoral b.indirect inguinal c.direct inguinal d.spigelian - baby delivered by caesarean section. 2. a pt with lymphadenitis & sinus discharging white caseous there are 100 multiple choice questions could not remember 18 material. the most likely diagnosis is? multiple choice questions…. 3. the most common cause of pid is? a. chlamdia b. tb c. e coli 1. patient had moon face, buffalo hump, abdominal stria. 4. which of the following enzyme rise first in mi? a. what is rx & name at least two d/d a. ast b. ck-mb c.ldh d. streptokinase b. name two ivs to confirm diagnosis c. name two radiological ivs which can be done in this case 5. flaccid paralysis in nepal should be reported to a. polio eradication centre b. kanti bal hospital c. health ministry 2. 50 yrs old man with urgency frequency nocturia undergone turp 6. the most important clinical diagnostic feature of kwasiworkor is b. what is tumour marker for ca. prostate? a. edema b.flag sign c.weight loss d.neurological dysfnction c. what are two sexual complications after prostatectomy? d. what are changes noted in bladder when there is bladder outflow 7. a lactating woman with 6 month old child has amenorrhea after deliver. she wants to use contraceptive. what must be advised? e. name two drugs for bph a.ocp b.barrier method c. urine test for b-hcg 3. patient complaining of whitish vaginal discharge. 8. the most common cause of condctive deafness in children? a. what is ‘4' differential diagnosis a.csom-aa b.csom-tt c.ome b. diff. between fungal & amoebic discharge c. what ivs will you do to confirm it? 9. the most common cause of unilateral foul smelling nasal discharge d. what is treatment? a.foreign body b.sinusitis c. adenoids d.asom 4. child 2yrs of age, maroon coloured faces & had agonizing pain, 10. which of the following must be given to a lactating woman? raise legs above abdomen. a.iron b.calcium c.folic acid d.vit b a. what is diagnosis? b. which organism is responsible for this? 11. an adult with # shaft of femur, skeletal traction is done by c. what is treatment? 5. g2p1, at b.p of 150/100mm hg pulse 80 10/min, at 37 wks 12. a pt with a h/o # neck of femur came with a c/o pain after 2 yrs. a. what drug'll you give? a.avascular necrosis b. what are signs & ivs of this? c. when'll you discharge patient? 13. which of the following is not an aids criteria? d. what advise will u give if b.p is controlled? & if b.p isn't a.fever b.wt loss >10% c.cough d.diarrhoea e. what is drug of choice if it's associated with convulsions? 14. a 21-yr-old boy came with a c/o pain and tender abdomen. x-ray showed gas under diaphragm. cause? 6. patient brought child with respiratory distress. a. appendicular perforation b.peptic ulcer perforation a. what're signs of respiratory distress? b. what're confirmatory signs & dd? 15. a 2-yr-old child with respiratory rate >45 and indrawing chest. c. what are ivs? d. what is treatment? a.pneumonia b. severe pneumonia c.common cold 7. patient had pain abdomen, with gb stone radiating to back, no 16. a child with uncomplicated rupture of tympanic membrane. what aggravated while lying in supine position. a.steroid and antibiotic drops b.iv antibiotics c.wait and watch a. what is diagnosis? d.immediate repair b. what is it due to? c. what are diagnostic criteria? 17. which of the following reduces progression of the severity of a.oral b-agonist b.oral b-agonist+steroid c.smoking cessation 8a. what are major manifestations of rh fever? b. which're the joints involved name 4? 18. spider nevi is seen in c. what is least time for involvement of joint? a. hand b. brain c.trunk d.palms nmcle 2007 sept 22 (asoj 5, 2064) 19. which is used for the radical treatment of malaria? total marks: 240 a.chloroquine b.quinine c.primaquine time: 12:00-2:00 pm (2hrs) 20. rigor mortis starts from total mcqs:100 (100x2=200 marks) a.lower jaw b.upper eyelids c.lower limbs total sqas:8 (8x5=40 marks) 21. term pregnancy, normal vag delivery, still birth. cause? a. dm mother b.iugr 44. menorrhagia mostly associated with 22. which of the following is transmitted by water and air? a. ca ovary b.fibroid uterus c.ca uterus a.polio b.typhoid c.hepatitis 45. a driver having erythematous plaques with silvery scales. cause? 23. a man from terai with non-pitting edema. the most likely cause? a.filaria b.polio c.dvt d.nephrotic syndrome 46. a 44-yr-old male patient with facial assymetry. on p/e touching the 24. early morning sticky, red eye. cause? cornea of either eye with a cotton swab results in blinking of only one a.conjunctivitis eye. he states that he feels the cotton swab touch in both eyes. 25. diagnostic accuracy of a test is determined by a. facial nerve paralysis b. trigeminal nv paralysis c.occulomotor nv a predictive value b.specificity+sensitivity 26. a mother at a health camp brought her 2-yr-old malnurished child 47. aciduria occurs in who had white patch on eye. cause? a.met acidosis b. met alka c.resp acid. d.resp alkalosis a. cataract b.xerophthalmia 48. microcytic hypochromic anemia. which test? 27. infant mortality rate in nepal a.schilling test b. hemoglobin electrophoresis c.g-6pd test a.41 b.50. c.98 d.64 49. a woman in ravibhawan complains early morning sneezing, nasal 28. which of the following is acynotic heart disease? obstruction and rhinorrhoea for a long period. o/e nasal turbinates are a.tof b.tricuspid atresia c.tga d.vsd without pulmonary hypertension pinkish and edematous. cause? a. vasomotor rhinitis b. allergic rhinitis c.nasal polyps 29. in tof there is vsd, pulmonary stenosis, rt ventricular hypertrophy, 50. a child with radioulnar #. which anesthetic agent used? a.asd b.overriding aorta c.coarctation of aorta 51. which anti-hypertensive agent can be given during pregnancy? a.produced by pituitary gland b.ejection of milk a. alpha-methyl dopa 31. umbilicus is supplied by 52. a patient with previous reducible hernia, now presents with pain, tenderness and irreducibility. cause? a.obstruction b.strangulation a.supported mostly by broad ligament b.supplied by hypogastric nerve a.mental retardation 33. the most common tumor of testis is? a. seminoma b.teratoma c. lymphoma 54. cross transfusion means? 34. att contraidicated in pregnancy? a.inh b.rifampicin c.pyrazinamide. d.streptomycin 55. a 6-week-old child with non-billious projectile vomiting. cause? a. hyertrophic pyloric stenosis b. dodenal atresia c. intussuseption 35. att causing uremia and gaut? a.inh b.rifampicin c.pyrazinamide. d.ethambutol 56. commenest complication of chicken pox? a.secondary infection b.meningitis c.pneumonia 36. febrile convulsion in a child is best treated with a.phenytoin b.diazepam c. ethox d.phenobarbital 57. a patient with gall stone suddenly develops pain abdomen, tachycardia and shock. cause? 37. antibodies detected in blood in typhoid a. acute pancreatitis a.1st week b.2nd wk c.3rd wk d. 4th wk 58. fluid filled lesion <1 cm diameter. 38. obese dm type 2 pt. best drug? a.macule b.papule c.vesicle d.wheal a. sulphonyl urea b.metformin c.insulin d.rosiglitasone 59. which sign in appendicitis? 39. dm pt with bp 160/95 mmhg, which anti-hypertensive is best? a.iliopsoas b.murphy sign a.nifedipine b.ramipril c.methyldopa 60. few hours after subtotal thyroidectomy a pt develops dyspnea. 40. a child with cough for 1 month and there is subconjunctival hemorrhage. diagnosis? a. hematoma b. rln paralysis a.whooping cough 61. which investigation accrately diagnose pulmonary embolism? 41. which of the following is a proton pump inhibitor? a.contrast ct b.ecg c.chest x-ray a.omeprazole b.cimetidine c.ranitidine 62. how to differentiate chronic bronchitis with asthma? 42. a pale looking farmer from biratnagar came for checkup. o/e he a.history taking b. chest x-ray was anemic but other findings were normal. cause? 63. rashes in measles starts from a.face b. trunk c.limbs 43. # of a bone into >2 pieces a.compound # b. communited # c.simple # 64. cause of neurocysticercosis? a.t solium b. t saginata c.ascaris 64. a pt with red eyes and intropion of few eye lashes. cause? a.trachoma b.glaucoma. c. cataract 65. height of a child doubles in 8. smoking male complains of right leg pain with black patches of a.4 yr b.5 yr c. 3yr d. 2yr skin of right toe and left calf pain when he walks. 66. child with fever and headache. csf: n 90% glu-dec,pro-increased. b. what is intermittent claudication? c. what is rest pain? a.pyogenic meningitis b.viral meningitis c. enceplalitis nmle questions 22th dec. 2007 - 27-12-2007, 04:16 pm 67. a pt fell from height and got # calcaneum. which other # is usually most common parotid gland tumour >> mixed (pleomorphic) associated with it? a. spinal # b. skull# which of the following vaccine is live attenuated >> mumps measles 68. how do you check xii cranial nerve? a. ask to protrude the tongue b.ask to say 'aah' c. ask to bend the neck which of the following is the complication of diptheria >>myocarditis 69. a pt with a h/o appendectomy develops distended rigid tender in measles rashes starts from >> face trunk hand abdomen. on x-ray multiple air-fluid levels. cause? a. intestinal adhesion which of the following is best method for the study of rare disease>> case control cohort experimeantal 70. an agitated adult man, couldn't sleep properly at night, thinks he is superior to his colleagues, quarrels with coworkers, makes new ideas. illusion is >> false interpretation of stimulaii anti-psychotic drg is indicated in >> schizophrenia anxiety diaorder 71. a woman with white curdy discharge per vagina. cause? a. moniliasis b.chlamydia c.trichomonas a child with odema an wt 70% >> kwasiyokar marasmus kwashiwokarmarasver short question answer a child with ht 90% wt 80% with wide wrist jt, bowin legs >> shunted retarded rickets 1. a carpet factory worker living in a crowded place presents with chest pain, evening rise of temperature, and mild weight loss. koplik spot is seen in >> typhoid measles mumps. b. investigations you want to order? a child with swellinnn on rt side of chick with tresmus, a student in his c. 2 treatments for this diagnosis. class is also sufferin from the same diseae >> mumps, parotid tumour 2. a man presents with acute chest pain for 1 hour. on ecg there is st 60yr male with horseness of voice n palpable ly node on elevation >1mm. supraclavicular ly node >> ca. larynx , recurrent nrv palsy, vocal cord b. investigations you want to do? c. how do you manage this patient in a zonal hospital? most common cause of infection in pp fever >> clamydia 3. a newly married woman with 2 months history of amenorrhoea most commonly affected in pp sepsis>> endomentrium salphinx presents with vaginal bleeding. a. d/d of this condition b. write non-invasive investigations 21 ??yrs male with vomiting , visible peristalisis lt to rt , suc.splash + c. how do you manage? ,>> small bowel obs., grastic outlet obs., large bowel obs. ?? yr person sputum -ve tb on att now becomes sputum +ve after 2mt 4. a child with fever, headache, vomiting, rashes on lower limbs. of therapy >> cnage to cat 1 chang to cat 2 . moutoux test is best read after >> 12 24 48 72 hrs (48) b.write 2 other tests which support your diagnosis 2 mth old child can do >>> .from motor developement c. causative organism? 100ml of breast milk contains how much cal. >> 47 57 67 77 (67) d. drg of choice? t/t of toxic nodule >> surgery medical radio iodine. female on pp period with red hot tender rt breast >> i n d, aspirate to 5. a woman complains of fever on 4th postpartum day for 2 days. confirm brest absess, flucloxacillin an f/u after 2 days, infectivity of hbv is indicated my >> hbeag, igmhbv b. what 2 informations in this case supports your diagnosis? t/t of choice of t.solium > py, meb, alb, (praz) c. how do you care this patient? opth. neonatarumi s caused by >> ghonorria, stap, strep pt with couldnot see far n near obj with pain eye>> glaucoma, presy. , 6. a toxic looking child presents with stridor and drooling. ??t/t of iritis >>atropin . with neoplasm usually venot mets at time of dia. > ssc, rcc, malig. b. 3 d/d of this case? melanoma, basal cell ca. c. which organism is responsible for this condition? gomet insertion in >> glue ear, csom, asom. d. drug of choice? ext. pyramadal rxn >> haloperidol 7. a lactating woman presents with severe breast pain and swollen tender mass in the outer quadrant of the breast. not adr of ketamin>> halucination, dissociative ana., inc. iop, iodine day >> ???? a;bronchiectasis which doesn't cause empyma in child >> stap, strep, myco, n. c;left to right shunt a clinical que from rheumatic hrt disease ?? d;cystic fibrosis compiled by dr. jj a clinical que from ms?? 6;a pt with organophosphorus poisoning is being treated by atropine comprehension ques. each carry 1 marks in the ward;his both pupils are dilated ;develops fever and is tachypnoiec ;suddenly he develops convulsion;what will you do;;;;;;;;;;;;;;;; common cause of meningitis >> h inf a;start pralidoxime csf finding >> >100 wbc with neutro indominent b;give iv midazolam t/t >> cefuro. c;give phenytoin common compli>?? d;stop atropine and watch one i can't remember 7;a 35 years female has solitary thyroid nodule;what will you advice? a;antithyroid drugs genetic abn > 46 x 45x 46 xxy 45xxy common associaeed contd > vsd asd co. of aorta pda common associated kidney disorder > horse shoe kidney t/t >growth hormone not found > ht tall, ht short,web neck . 8;a 40 years male has developed bronchial asthma;what will you a;terbutaline nebulizer in chest >2mm st elevation >> ant mi ,ant septal mi. , massive ant mi b;steroid therapy drg for ?? reperfusion >> streptokinase?? , aspirin, gtn. coz of death in 1st 24 hr >> venti arry, ccf. d;montelukast therapy which drg comb. prevent decrease pain n prevent remodelin >> o2 morphine b block, aspisin ace inhibit. 9;1 gm hemoglobin combines with pansystolic murmur coz?>> vsd, mr ,ms to calculate % of burn <rule of nine> fluid requirement (parkland formula) 10;which of the following antibiotic is c i in suspected tb pt? nmcle poush 7 th 2064 (22 decmber 2007) correct answers are given in bold italic letters. please help yourself regarding unanswered questions and if any doubt about high lighted 11;paronychia is a infection of b'pulp of the nail 1;which of the following is true c;apex of the nail a;acidic drugs are best ionised in acidic medium d;base of the nail b;gentamycin is given i;v as it is unionised in git 12;a dehydratd pt has fever;which of the following complication compiled by dr. jj a;mis interpretation a;respiratory acidosis b;metabolic acidosis (please help yourself too) c;respiratory alkalosis 3;b blocker should be avoided in d;metabolic alkalosis 13;which of the following is not the macrovascular complication of b;cerebral thrombosis 4;a pt; with sputum negative pulmonary tb is treated with category 3 c;peripheral neuropathy regimen of who for 2 months and he became sputum positive at the end of 2 month;how will you treat this pt; further? a;stop this regimen and start with cat=1 b;stop this regimen and start with cat;2 c;continue this regimen and recheck sputum at 5 months d;switch to 2nd line drugs 5;clubbing is not found in 15 a pt; came with alcohol poisoning in the emergency department where you are medical officer;what will you do b;give 3;5%hypertonic saline c; spontaneous recanalization of cord c;methyl alcohol (please help yourself too) 25; a 27 years old male patient has found patent processus 16; a poor farmer from phoolbari vdc near dhangadhi came in the opd vaginalis;what could be the possible complication/diagnosis;; without slipper and proper dress with the h/o loss of appetite ;weight a;inguinal hernia loss and is icteric on examination;what could be the possible diagnosis-------- d,testiculour tumour 26;parotid tumour-most common d;hookworm infestation 17;diabetes is suspected if the fasting blood sugar is above 27;which of the following develops in the first post operative day? 18;human milk provides b;57kcal compiled by dr. jj 28;a pt; suddenly presents mass outside the anal canal with some blood streaks in the stool;after 2 hrs later while defecation similar c;67kcal (please help yourself too) events occurred;what could be the most possible cause;;;;;;;; 19;which of the following is not macrolides;; d;prolapse of the rectum 29;a pt with signs of dehydration and altered sensorium admitted to the emergency department with serum sodium level 115meq/l;which fluid will you prefer;;;;;;;;;;;;; 20; a person with damaged mitral valve develops endocarditis;the most common organism is;;; b;staph epidemidis (please help yourself too) d;3=5%hypertonic saline compiled by dr. jj 30;a 32 years old patient starts non bileous projectile vomitting with 21;a 26 years male starts micturition;what will be the pressure inside contains of food particles and has distended urinary bladder? a bdomen with visible peristalsis and circum splash is admitted in your ward;what could be your most probable diagnosis'' ' a:gastric outlet obstruction b;small intestine obstruction c;large intestine obstruction 22;a 6 moth child came in the opd with posterior triangle swelling in d;esophageal stricture the neck;what could be the possible cause? a;thyroglossal cyst 31;a 2 months child can;;; a;hold head on ventral suspension c;cystic hygroma b;has protective reflex against external stimuli d;thyroid swelling 23;a lactating mother came to your opd with right breast swelling;on 32;a child has 70%body wt of his expected wt at that age with examination found red;tenderness and swollen;;;;suspect breast oedema;;;which one is true what is your next step a;aspirate and confirm abscess (please help yourself too) b;give antibiotic and ask her to come after 3 days d;undernutrition c;incision and drainage d;confirm abscess by ultrasound 33;a moderately dehydrated child is in your hospital;what is the fluid requirement for first 4 hour 24;vasectomy-most comman complication b;varicocele compiled by dr. jj a;placenta previa 34;a 8 years child with puffiness of the face and periorbital oedema b;abruptio placenta which later develops in lower extremities;;what is the most probable a;nephrotic syndrome 45;anti tb drug avoided in pregnancy c;cushing syndrome d;downs syndrome 35;nephritis-false 46;a 42 + weeks pregnant lady with abdominal pain and not fully dilated cervix has done arm ;what will be your next step a;augmentation by oxytocin b;induction by oxytocin 36;b- thalassemia-choose the correct one c;apply cerviprime gel locally a;both a and b chain increased compiled by dr. jj d;give prostaglandis for induction 47;koplik spot is seen in b;both a and b chain decreased c;a chain increased and b chain decreased d;b chain increased and a chain decreased (please help yourself too) 37;adolescence period 48;a 5 years old child has difficulty in opening the mouth; unilateral parotid swelling and few days before his classmate had also developed the similar swelling;what could be the most common 38;menorrhagia is a common feature of;;;; c;ovarion tumour 49;most common complication of diptheria a;peritonsiollar abcess 39;uterus nerve supply b;oral thrush compiled by dr. jj a;hypogastric plexus c;myocarditis (please help yourself too) 50;measles rashes first appears in 40 uti in pregnancy--;safe drug b;nitrofurantoin 51;cysticercosis –drug of choice 41;pid-commenest causative agent a;pyrantel pamoate 52;a pt with plasmodium falciparum malaria came to your opd;what 42;a pregnant lady with blood group a positibe needs immediate blood will be your initial treatment transfusion after delivery;but that blood group is not found;what will a; tab;chloroquine base 600 mg followed by chloroqine base 3oo mg in next 6 hr and 300 mg for 2 more days a;give a neg; blood group b;iv quinine with 10% dextrose b;give o negative blood group c;give ab positive blood group d; give b poitive blood group 53;je is transmittd from 43;pap smear is taken from compiled by dr. jj a;culex mosquito c;anopheles mosquito a;anterior fornix b;lateral fornix 44;38+w pregnant lady has develod painless bleeding per vagina without abdominal pain;commonest cause 55;rare causes are studied under 66;in scabies there is a;descritive study 67;paucibacillary single lesion tuberculoid leprosy is best treated by a=the numerator includes child of age less than 28 days b;less than 7 days c;less than 1 year c;amphotericin b d;less than 2 months compiled by dr. jj 68;a man has habbit of pricking hairs in beard shaving area and the hairs prick easily 57;live attenuated vaccine a;psychiosis barbie 69;eps is causd by 58;hiv by needle prick transmitted d;phenyl hexidine 70;antipsychotic drug is used in 59;ocp- mechanism a;supress lh by negative feed back mechanism and prevent s ovulation b;anxiety disorder c;schizophrenia compiled by dr. jj 60;life long immunity is achieved by 71;a pt on tricyclic antidepression drug has taken overdose;what could be the most common complication 61;ketamine-false d;cerebral oedema 72;a pt with liver disese sice last 3 months is under treatment and he finds himself that he is not improving ;he is in home stay for long time;now he is irritatve and thinks he is not going to be alright and is sad;what has happened to him 62;which of the following is paired cartilage a;generalised anxiety disorder a;thyroid cartilage b;cricoid cartilage d;organic mental disese 73; in a case of diabetic retinopathy the earlier sign is: 63;sinus not present at birth a;retinal hemorrhage compiled by dr. jj 74;iridocyclitis patient; what will you give 64;gromet insertion is done in a;glue ear(ruptured tm) d ;acetozolamide b;fungal infection 75;a 60 years old man with h/o gradual loss of vision with mild pain d;foreign body in external ear and h/o headache ,blurring of vision ;what is your most probable 65;60 years male presented in your opd with the complain of hoarseness of voice and h/o smoking; most probable diagnosis is a;b/l recurrent laryngeal nv palsy c;acute glaucoma b;carcinoma of the larynx d;conjunctivitis d;fb obstruction 76;age estimation can be done by: b;acute rheumatic fever c;septic arthritis c;bone calcification d;osteoarthritis d; compiled by dr. jj 89;treatment of paraphimosis b;multiple puncture of glands 90. clinical question regarding referred pain from chest to shoulder 78;cobra venom is 91;onwards could not remembered ***********best of luck************ if any querry, contact: 79;which of the following is developed from shaft of the bone b;osteoid oste0mas compiled by dr. jj 80;compartment syndrome is more danger when it is in a;lower end of femur b;upper end of humerus c;lower end of tibia d;lowe end of humerus a= a 4 years old child presented to you with turner syndrome 1' what will be the xenotype of this patient? 81;orthopaedic emergency a;septic arthritis b;fracture neck of femur c;fracture skull d;rheumatoid arthritis 2'which of the following findings is present in this patient? 82; fracture of shaft of femur --treatment in child of 2 yrs a;abobe knee plaster b'coarctation of aorta c'endocardial cushion defect 3'which of the following is associated with turner syndrome? 83 .which of the following investigation is ordered for maximum b'hoarse shoe kidney infective period in hepatitis- b patient? c;minimal change disease 4'which of the following is fase? compiled by dr. jj 5;treatment of turner syndrome? a;growth hormone 84;which of the following disease is benign? a;basal cell carcinoma b;malignant melanoma c;squamous cell carcinoma b;9 month old child with features of meningitis 1most comman 'causative agent ? 85;iodine day in nepal a. b; c; d ??????help yourself 86;ophthalmia neonatorum -most common organism 2;true regarding csf findings a;cell count 650 with mostly neutrophils d'cell counts 200 with mostly lymphocytes 87.10 years old child with multiple large joint pains for two 3;true regarding brudgkisky sign weeks;most probable diagnosis is a;neck pain while passive flexion of neck b;flexion of knee while bending neck compiled by dr. jj 3; what is the fluid requirement for 24 hour of this patient c;pain on thigh while bending neck d;neck stiffness 4;regarding brain abscess at this age which one is not common? 4:true regarding burn a;bubbling is seen in superficial burn b;pain is more in more depth burn c;crt is decreased in superficial burn 5;treatment of above case 5;which one of the following must be monitered in this patient b;crystalline penicillin compiled by dr. jj 22 probable questions c= a case of cholelithiasis_- _-_ 4; a patient came 4 hours after the burn time how will you calculate 1most common organism associated with infection fluid requirement a; generally 50% fluid should be given in first 8 hours c'staphyllococcus b;rest 50% fluid should be given in next 24 hours c;in this case 50% fluid should be given in next 4 hours 2;immediate comlication of this patient 5;fluid of choice b;acute cholecystitis d;acute pancreatitis d;hypertonic saline 3;black colour stone is due to e;a fifty years male pt with myocardial infarction 1;ecg changes are >2mm st elevation in leads v1 to v6 ;what is your 4;can happen except b;posterior wall mi rsion and st elevation in lead v1 to a;anteroseptal a;du perforation c;extensive anterior wall mi b=gall stone ileus d; inferior wall mi 2;drug for reperfusion compiled by dr. jj 5;all are true except a;10% gall stones are radio opaque 3'most common cause of death in 24 hr? b;90%gall stones are detected by usg a vent;;arrythmia c:90% gall stones are radioluscent d;10%gall stones are detected by usg 4 pansystolic murmur d;60 kg man with h/o burn presentd in emergency 1;how will you access the severity of the burn a;depth of the burn (please help yourself too) b;surface area of the burn c;weight of the patient compiled by dr. jj d;sex of the patient 5; streptokinase is given; further treatment by(to decrease remodelin) 2;the pt has lower left extremity burn what is the % of burn of this patient? d;ace nhibitor+b- blocker+aspirin f;a case of type 2 dm 1'obese pt with h o generalised pruritus and serum creatinin level 2=5 b)changes in heart in ac. rheumatic fever mg%;what will you give c)why she came to hospital at this time? c;metformin+gliactazone 4) clinical features of hiv in children. d;metformin +sulphonylurea 5) major criteria of rheumatic fever? 2.which of the following is not the macrovascular complication of 6)30 yr f came with pain right hypochondrium on examination serum Page bilirubin 4mg%,multiple stones in cbd, dilated cbd 2;cerebral thrombosis a) write its surgical t/t 3;peripheral neuropathy c) complication due to lft 3;true regarding type 2 dm a; it is associated with addisons disease a) difference between external and internal inguinal hernia b;it is autoimmune disease b) femoral vs inguinal hernia relation of hernia sac and spermatic cord c) omentocele vs enterocele 8) criteria for normal labour 5;dm is found in a=thalassemia pt 9) risk factors for ca cervix b- iron deficiency anaemia pt c;aplastic anaemia pt 10) 102 f temperature ,vomiting ,rashes in legs d;sickle cell disease a) investigations compiled by dr. jj 11) what are the five signs of atropinization in case of op poisioning? what advice will you give on duty sister while managing op poisioning case? g=38+ weeks pregnant lady presented in emergency with vaginal bleeding and mild abdominal pain;she has given the h/o vaginal 12) investigation modalities in case of ca. breast . bleeding 8 hors back management in case of ca breast. 1;what is your most probable diagnosis 1. caustive organism of impetigo.>group a streptococcus. 2. dose of acyclovir for herpes zoster infection.> a) 800 mg five times a day 2 pregnant lady at term;on examination no fhs heard, head at station +2 with late decelleration;;;delivery by 3. the most abundant quantity found in clostrum milk is d;normal delivery a)water b)fat c)protein h;2 month pregnant lady withc/o vomitting' severe right iliac pain 4.commonest cause of dementia. with right lower abdominal pain and rebound tenderness is presented alzamers disease 1;what is your most probablke diagnosis? .low pitch mid diastolic murmur. a;acute appendicitis a)mitral stenosis. b)tr c)as b'ruptured ovarion cyst c;ectopic pregnancy (please help yourself too) 6) which drug is contraindicated in ccf? d;-intetinal obstruction compiled by dr. jj a)propanolol. b)ace inhibitor c)ccbs 7.hematuria with rbc cast. 2;retention of urine in pregnant lady;most common cause a)nephritic syndrome b)nephritic syndrome c)arf d)crf a;retroverted uterus nmle questions chaitra 3rd(march 17, 2007) - 17-03-2007, 04:32 pm 8.t wave inversion in ecg is seen in. a) hypokalaemia b) hypothermia c) hycalcemia 1) causes of massive spleenomegaly? 9. which wave is seen in ecg in hypothermia? 2)signs of diabetic retinopathy? a) j wave b) delta wave c) tall t wave 3)20 yr f/ pt came to emergency with sorthness of breath. according to 10. commonest surgical cause of cushing syndrome? her husband she is pregnant for 2 months . on examination , pulse a) putitary adenoma b) ectopic acth. 100, b.p. 90/60mmof hg, pulmonary oedema opn auscultaion . (please find the answer yourself too) 11. normally anterior fontanel is close in investigation bilirubin is raised and impaired lft.what a)18 months b)12 months c) 8 months drug you want to stop? 12. you a re going to drain an abscess under ketamin, which rifampicin b)pyizinamide c)streptomycin d)ethambutal condition you want to rule out? a)hypertension b) diabetes c) smoking 30. which is true about hansen's disease? a)hypoaesthesic patch, afb +ve. thickened nerve. 13.12 years old child has got high grade fever pharyngitis and b)hypopigmented patch,afb+ve, thickened nerve sore throat. after 4 days fever subsides and measles like rashes appear what might be the cause? 31. a patient of multibacillary leprosy was under anti leprotic drugs.he developed a red discoloration of skin, a)scarlet fever b) measles c) erythema sabitum d).enteric fever which drug is causing it ? a) rifampicin b) dapson c) clofazimine d) minocycline 14. spalding sign denotes a) intrauterine fetal death b) ectopic pregnancy c) molar 32. a 30 years female patient with normal vaginal delivery with episiotomy came to the sopd with the h/o pain during defecation and episode of bleeding on stool 15.which is the shortest pelvic diameter? what investigation you want to do? a)digital rectal examination b) sigmoidoscopy c) proctoscopy 16.caloric test is done for?>vestibulo coclear d) gently apart the buttock and see. 17. when a person first time exposed to the disease in the 33. a 25 years male patient has resting tremor. on epidemic areas is called?> investigation he has normal t3 and t4. what is your a) index b) primary. diagnosis? > ? 18. cause of b/l partial loss of vision 34. a female patient on examination has b/l thyroid lump a)temporal lesion b)glaucoma c)optic glioma d)pituitary .what investigation you want to do initially? (please be sure about c and d) usg b)fnac c)tft d)biopsy 19.a female of 26 years is taking oral hypoglycaemic drugs. she 35. which one is true about bcg vaccine? came to the gopd with the c/o vulval itching and discharge. what given subcutaneously b)prevention against tubercular might be the cause? meningities c) life long protection against tuberculosis.d) a)fugal b)bacterial 20.antemortem death due to the drowning.what is the surest sign. 36. a 23 years female patient came with labour a) oedematous lung. b) goose skin. c) diatoms pain.medical officer did p/v examination then he found cervix fully,head at the level of ischial spine ani 21.what is the visual acuity for the blindness according to the pusatile cord. what you will do next? a)ceseration section b)forcep delivery c)ventouse d) wait and a) 1/60 b)15/38 c)6/60 d)6/6 22.6 years boy came to the ent opd c/o pain ear.o/e there is tender 37. a 9 years boy has painful swelling on the parotid on pressing the tragus.what is yous diagnosis? region. which of the following vaccine may prevent a) asom b) acute otitis externa c) mastoiditis d ) csom a)mmr b)bcg c)opv d)dpt 23. antipsychotic drugs is used for which condition?> nmcle 7th asad 2065 24. what is the commonest sign of schizophrenia? a) visual hallucination b) auditory hallucination 1] drug of choice in uti with pregnenacy a b nitrofurition c d 25. what is the weight of termed sized uterus? a)500g b)900g c)1500g d)2000g 2] a two yrs child with running nose , sore throat , pharyngitis 26. a women of vaginal; delivery on 3rd ppd developed a mild developed sub conjuctival hemorrhage , what is the most likey fever. what might be the cause? provisional diagnosis ? a) uti b) puerperal sepsis c) breast engorgement 26. which is true about hospital acquired infection? a)resistant to many antibiotic b)drugs for hospital acquired 3] pleomorphic rashes seen in infection is similar to the drugs for community acquired infection a measles b mumps c chiken pox d small pox given by good hospital c)hospital acquired infection is not seen 4] which of the following drugs is mood stabilizer ? 27. sensitivity means ? a chlorpromazine b promethazine c carbamazipine d proclormapazine 28. mode means?> commonly occuring value 5] anxity neurosis 29. 32 years male ,teacher by occupation., is taking att drugs. he develops pain on right hypochondrium. on 6] thought block seen in 28] treatment of paraphimosis 7] 30 yrs adult patient has irritating eye , watering discharge with a circumcision b c d periorbital swelling , the most common cause is 29] endometritis a pyogenic conjunctivitis b viral conjunctivitis c allergic conjunctivitis 30] 40 yrs old lady in usg fibroid uterus with asymtomatic 8] a molnutrient patient develops sudden resent onset of opacity of the a hysteria b myomectomy c hysterectomy d hysterectomy with eye came in eye camp, the most cause is 31] most common site of breast ca 9 ] ectopic pregnancy 5 yrs a upper and outer b c d 32] young man with rta and the most dangerous complication 10] malaria hypnozoites a head injury b middle cerebral artery c post 33] the most comman site for lumber puncture 11] cin staining 34] deficency of ca+ causes in child 12] uterus support a osteoporiosis b ricket c osteomalacia d 35] tetany is caused by a decreased ca+ b c d 36] alcoholic patient with malena and hematemesis a iv saline with blood b rl c d 15] fast putrifaction in which poisoning 37] palmar erythmia is seen in 16] mean , mode median a liver disease b renal disease c d 17] consent is not taken in 38] the most comman cause of alopetic areata a age estimation b rape case c postmortem 18] pyogenic meningitis 39] hypotonia , atexia and tremor seen in a celebral lesion b c d pons 19 ] negri bodies 20] supracondylor fractor 41] drug not given in petit mal seizure a clonazepam b c d 42] scanty, foul smelling with discharge seen in 21] septic arthritis , the most common causative agent 43]lady with sedation 22] local anaesthesia moa a anxiolytic for sedation b c d 23] 70 yrs old , tibial fracture - which anesthesia 44] antidepressive drug causes a cardiac arrhythmia b c d 45] 9 month child with respiratory rate 50/min , fever and dyspnea 24] 5 yrs old child with nasal block , irritant nose , the most common a urti b pneumonia c d 46] 2 and half yrs child having visual impairment 25] drug c/i in pregnancy a tetracycline b c d 47] measeles like rashes seen in 26] 50 yrs old man shows horseness of voice since last 3 yrs rosea infantum b c d 48] syphilitic ulcer in vulva with painless treatment is 27 ] rda of protein a undermined b everted c d 50] late mid diastolic mumur is heard in 16.mood stabilizer 17.multi drug resistent 51] the commanest congenital heart disease is 18.sars- causative virus 52] clinical question regarding rickets 19.glomerulonephritis is caused by a.plasmodium mamariae 53.question regarding polio, neuroppathy,gbs,transverse myelitis 54.fluid given in antrum carcinoma 22.cpr ratio in child 55.who definition of blindness 56.questions regardig all in child 23.atropine c i in 24.steroid c i in 1.clinical questions regarding cholelithiasis 25.mushroom poisoning rx 2. clinical questions regarding myocardial infarction 3. clinical questions regarding burn 4.megaloblastic anaemia 28.lichen planus 29.alcoholic male 7.retention of urine in early pregnancy 8.ectopic pregnancy nmcle chaitra 02-2064(march 17th-2008) mcqs-2marks each 33.hyperemesis gravidarum 34.recurrent miscarriage 2.commonest primary bone tumour……… 35.acute severe asthma 4.nifedipine-most common side effect 37.perforated uterus 38.# neck of femur 39. bag and mask ventillation 40children …. common 7.septic arthritis 8.septic arthritis 42.kala zar drug 9.meningococcemia 43.hida scan is done in 10.pyogenic bacterial meningitis-csf findings 45.pneumonia cut off point in child 47.influenza virus 14.commonest ca larynx 79.streptomycin- s e 80.rigor mortis starts from-periorbital region 81.breech presentataion 82.no urine output………… 52.bartholin"s cyst a.marsupilization 84.hepatitis-regarding pathology 53.24 yrs 6*6cm2 54.post operatve dyspnoea-common cause 86.oxytocin-regarding functions a.haematoma formation 55.rectal cancer ssaqs-5 marks each 56.bone marrow transplantation 57.normal titre in typhoid 2.diarrhoea and dehydration 58.interstitial keratitis 4.rectal bleeding 60.atypical pneumonia 7.ovarian tumour 8.megaloblastic anaemia nmle questions 9th kartik 2065 - 27-10-2008, 07:07 pm 61. ethambutol dose in tb? well i don't remember all the questions but here are some of them: 62.monilial infection-drug 1. some bowel surgery was done. after a week, what would be the 63.acute flaccid paralysis d/d symptom, if the abdomen was to burst? a. abdominal distension 64.frost bite treatment b. leakage of pus, secretions 65.hypothyroidism- earliest symptom 2. presence of ld bodies suggest 66.mitral stenosis-murmur 68.pulmonary embolism-medical emergency 3. a man with kalazar will die of a. spleenic rupture 69.highest refractive index in b. bacterial infection 70.uv prolapse-early prevention a.pelvic floor exercise 4. a mydriatic drug with no cycloplegic action b. phenylephrine a.urine output <.5 ml(1ml)/kg/hr 72.sigmoid volvulous 5. a patient taking tcas might have (tricyclic antidepressant) 73.stroke volume a. cardiac arrythmias b. severe headache 74.surgical emergency c. increased intracerebral pressure 75.pneumocystic carinni rx 6. a man working in cotton dusts will have ("brown lung disease" , 76.ampulla-commonest site for ectopic pregnancy 77.mucolytic drug 78.safer antihypertensive drug in preg 7. which one of the following is contraindicated in corneal ulcer? c. phenylephrine b. acute exacerbation of copd 8. which one is not associated with hypopyon? a. endophthalmitis 19. which is the commonest site for large intestinal polyps? c. corneal ulcer b. sigmoid colon d. transverse colon 9. if a person with acute congestive glaucoma develops blindness, it is 20. which has the maximum malignant potentiality? a. optic atrophy a. adenomatous polyp b. lens degeneration b. puetz jeghers polyp c. vitreous hemorrhage c. hyperplastic polyp d. corneal opacity i will post the rest later. 10. a person after head injury has diplopia, which nerve must have nmcle qsns of 25th oct - 28-10-2008, 04:45 pm i wrote some 90 qsns and i lost them.and i have no idea where did they go.anyways.i will write them again but this time shorter version.hope it helps!!! 11. which is the earliest sign of retinoblastoma? (cat's eye) 1) choice of contraception in young couple- ocp b. fungating mass 2)rectal polyp with high malignancy transformation- adenomatous d. corneal ulcer 3)painless pr bleedin bright red in colur- haemorrhoids 12. which of the following kidney stones can not be seen in xray? 4)convulsions less than 10 mins in 2 yrs child-febrile convulsions 5)fever with unconsiousness in 3 yr old child with non blanching rashes over lower abdomen-meningococcal meningitis 6)time of iucd insertion post delivery-after 6-8 weeks 13. a post menopausal women took estrogen and he developed side effects. if she wants to have an alternative treatment of pms, what 7)infertile couple first investigation-seminogram would you suggest? a. phytoestrogen 8)trichomonas vaginalis,rx-metronidazole b. antidepressants c. green leafy vegetables 9)lady with intense pruritus over vaginal region, type of discharge- 14. a son had a diabetic father. just to be in safe side, he went to have 10)laboring lady,suspected of face presentation, to confirm-vaginal his blood sugar checked. which one of the following would be the most diagnostic? a. blood fasting sugar > 7 mmol/l 11) putrefaction-fastest in earth and slowest in water b. blood random sugar >11 mmol/l c. plasma two hour post prandial sugar >11mmol/l 12)blue line in gums-lead poisoning d. glycosylated hemoglobin level > 7% 13)best way to identify living person-dactylography 15. an adult fell off a tree and broke his tibia which is a open fracture. what would you do after resuscitation and before actual management? 14)commonest ovarian ca in 45 yrs old-epithelial a. wash and start iv antibiotics b. just wash and apply cast 15)pph not controlled by massaging the uterus or uterotonic,next step- c. debride the wound and wash b/l iliac artery ligation 16)undermined edge ulcer- tb 16. a 27 year old male had fracture intracapsular right hip. how would a. apply skeletal traction and immobilise 17)young lady with difficulty breathing-thyroglossal cyst,brachial b. hip joint transplantation cyst, cystic hygroma,thymoma.search for the ans not sure about the c. internal fixation with multiple screws. 17. which one will have life long immunity once infected? 18)lady with solitary thyroid nodule with tachycardia and sweating,tx- 19)child with constipation,prolonged physiological jaundice and mental retardation-cretinism 18. a man has early morning couph and sob. he has mucoid sputum. 20)lactating mother should get - iron supplement what is the diagnosis? 21)breast feeding ci in - hep b 22)gastric juice secretion per day - 2500ml 51)4 yr old child with cyanosis of lip and nose - tof 23)calorie req of adult with sedentary lifestyle-2000kcal will post other qsns later.i gtg right now.hope it helps.please check 24)lady after husband's death sad n takes no pleasure in anything- the answers again some might be wrong!!!good luck re: nmcle qsns of 25th oct - 26-11-2008, 09:14 am 25)bells palsy-unknown etiology hi blotto.the pattern has been changed for this time.the qsns were all 29 26)abductor of vocal cord - post cricopharyngeal muscle mcq.no short answers question!!.not sure if the next exam will be of the same pattern or different though. 27) not related to submandibular gland- glossopharngeal nerve here r some more questions.it has been really long time though.almost 2 months.so i will jot down the ones i remember. 28) true about ocp-comes in 30 days pack as to maintain regularity 1.wat wud suggest pneumonia in a 2 year old child-rr of more than 29) false about ocp-vaginal monalisis is decreased 2.csf finding of raised wbc,decreased sugar and high protein-bacterial 30) puo can be due to-lymphoma 31) child with loose stool blood and mucous mixed- bacillary 3.presence of ld bodies - kala azar 4 cause of death in chronic kala azar - splenic rupture 32)live vaccine- bcg 5. cause of death in pt with chronic renail failure - hyperkalemia 33)37 weeks lady with decreased fetal movement,bestthing to do-ctg and fetal heart pattern monitoring 6.cause of death in a pt taking tca antidepressant- cardiac arrhythmias 34)recurrent abortion at 6 and 8 weeks,presented now at 2 mths.next 7.steriods ci in- corneal ulcer thing to do- usg 8.radiolucent renal stone- urate 35)abd pain,2 8 weeks preg, anemia and brownish p/v discharge- 9.hypopyon not seen in- scleritis 36)maintainance fluid for 30 kg child-70ml/hr 10.atropine ci in- narrow angle glaucoma 37)who setup in – 1948 11. sle is associated with- episcleritis 38)epidemic dropsy - argemone seeds 12.diagnostic of ra- involvement of small joints of hand 39)commonest tumour in aids patient - either kaposi or burkitt 13.person with head injury and raised icp and diplopia. nerve injured- lymphoma,please check it 40)diagnostic feature of child who is hiv positive - chronic diarrhea 14.commonest cause of end stage renal disease- dm 41)sound ampilification in auditory canal done by which structure-? 15.comonnest cause of sub-arachnoid haemorrhage: rupture of berry 42)moderate dehydration treatment-search the ans please 16.one qsn about most diagnostic test for dm- fasting blood sugar > 43)h/o fall of child in outstretched hand and is able to pronate and supinate-supracondylar fracture 17.commonest site for large intestine malignancy- rectum 44)immediate complication of neck of femur fracture - sciatic nerve 18.55 yr old man having morning cough presents with shortness of breath.he gives history of mucoid sputum- a/e of copd 45)labor with continuous fall innjur on the right shoulder,with pain and swelling in the supraclavicular region with sensation intact- 19.child with fever and no other symptons.o/e child is active and chest posterior dislocation of shoulder joint has b/l crepts, antibiotic of choice- amoxycillin 46)not an acquired sinus - either preauricular or urachus.not 20.cephalosporin of choice in pseudomonas- cefazidime 21.life long immunity post infection - mumps 47)fistula is - abnormal tract lined by epi tissue connecting two viscus 22.highest chance of malignant transformation- adenomatous polyp 48)oph neonatarum-n.gonorrhea 23.chromosomal abnormality in down's- trisomy 21 49)commonest cause of pid - chylmadia 24.open fracture of tibia,first step after resuscitation- debridement 50)bartholin cyst,tx of choice-marsipulisation 25.earliest sign of retinoblastoma- leucocoria 8)which match the best?? a:chlamydia:discharge with clump 26.young man with fracture of intracapsular fracture of hip joint,tx of b:gonorrhea:whitish discharge c:trichomonas:profuse discharge choice- closed reduction and internal fixation 9)60 yr patient with secondary uv prolapse and decubitus ulcer.what 27.radical tx of malaria-primaquine would you do?? a:lefort operation b:vaginal hysterectomy with pfr c:abdominal sling operation d:manchester operation 28. typhus caused by-rickettsia 10)32 yr female with completed family size with cervical 29.mydiatric drug with no cycloplegic action-phenylephrine lengthening??what is the treatment of choice?? a:manchester operation b:lefort operation c: abdominal sling operation d: vh with 30. surgical emergency-paralytic ileus 31.man with history of jaundice for 1 month,mild fever and pain abdomen- liver abscess 11)15 year girl presented with h/o prolonged 4 days of p/v bleeding.what is ur diagnosis?? a:dysmenorrhea b:menorrhagia 31.diagnostic of hepatitis b infection- hbsag c:dysfunctional uterine bleeding 32.all are life threatnening except-diaphragmatic hernia 12)30 yr old female with intermenstrual bleed.what is the most likely diagnosis?? a:cervicitis b: endometritis c: copper t insertion d: 33.child with h/o previous sore throat fever presented with joint pain and subcutaneous nodules- rf 13)a pt is intubated.what is the reliable sign u would like to check?? 34.commonest type of ca larynx- squamous cell a: capnography b: chest auscultation c: pulse oximetry 35.false about tropical splenomegaly syndrome- can reccur after 14)target value for tb control and case detection rate in nepal??.i couldn't remember options 36.c/i of tonsillectomy- haemophilia 15)measles vaccine is given ?? a: intradermally b:subcutaneously c:orally d: intramuscularly 37 blood test to be dearranged in obstructive jaundice- pt 16)which is not combined vaccine?? a olio c:measles d: bcg 38.cotton dust inhalation causes - byssinosis 17)what is true regarding bcg?? a: live vaccine b:killed c:toxoid 39.sign of wound give away in a woman with post cs scar and in labour- maternal tachycardia 18)in pelvic inflammatory disease a combination of metronidazole 40. sign of impending wound infection post surgery-serous and pus and which drug is given?? a:doxycycline b:ciprofloxacin c:penicillin discharge from the wound 19)a 5 yr child with fever stridor…has glottic edema.sitting with both 41.cause of blindness in acute congestive glaucoma- optic atrophy his hand forward.what is the most likely diagnosis?? this is all tht i can remember.sorry for posting it this late!! a:bronchiolits b:adenoids c: acute epiglottitis d: croup 20)a 4 year child present with fever ,rr 52/min b/l wheeze.he had nmcle chaitra 2065 questions - 09-04-2009, 10:40 pm similar past history.what is your diagnosis?? a: acute bronchiolitis b: bronchitis c: bronchopneumonia d:asthma maximum questions were clinical.i couldn't remember all .i m just writing summary of questions… 21)a 30 yr female presented with h/o …….impending eclampsia….which is the doc for this???a: nifedipine b: methyl dopa 1)rita 22 yrs old….with h/o…….flight of ideas,thinks superior to her c:hydralazine d: mgso4 collegue…what is the diagnosis?? ans: mania 22)which is least likely in eclampsia??a: headache b:epigastric pain c 2)what is true regarding hallucination??a:false belief b:false perception with stimuli c:false perception without stimuli bp>120mmhg d:blurring of vision 3)iv regional anesthesia .drug of choice?? a: lidocaine b:bupivacaine 23)a 19 yr patient presented with h/o mass on left side of floor of mouth.there is pain when he takes sour food which goes off spontaneously.on palpation there is 3*3 cm2 mass firm mass on left 4)a patient with h/o seizure.all are drug of choice as anaesthetic agent side on submandibular region.what is your diagnosis?? a:lymphadenitis b:ranula c:submandibular calculi except. a:etomidate b ropofol c: ketamine 24)one case from fibroadenoma .i couldn't remember question 5)60 yr old patient.spinal anaesthesia of choice. a:0.5% lidocaine b:0.5% bupivacaine c:0.5% bupivacaine heavy 25)a couple presented with h/o infertility for past 14 months.what is the first thing you would like to do?? a: semen analysis b: serum 6)which stain is used for treponoma pallidum?? a:gram stain b:giemsa gonadotrophin c:progesterone level d:tubal patency test stain c: dark ground d: ziehl nelson 26)a27 yr patient has h/o fall from tree.he present withpain on 7)regarding syphilis which is true??a: penicillin drug of choice in wrist.there is mild swelling….on examination he has tenderness on primary syphilis b: penicillin doc in secondary c: in primary and anatomical snuff box.what is the diagnosis??a:monteggia secondary not in tertiary d:in all stages b:greenstick fracture c:scaphoid fracture 27)a 25yr patient presentd with 3 days h/o fe ver throat pain.on examination there is inflammed tonsils congestion…with uvula nasociliary , lacrimal nerve shifted to left side .what is the diagnosis?? a:acute tonsillitis 49)xerophthalia treatment question b:parapharyngeal abscess c:peritonsillar abscess d:pharyngitis 50)innepal vitamin a supplementation programme done for which group??ans: 6months-5 years age group 28)one question from acute leukemia( blast cell >21% in pbs) 51)scleritis associated with which disease???ans:rheumatoid arthritis 29)one question from cll 52)what is most likely in multivalvular heart disease??ans:constrictive 30)a 45 yr patient presented with h/o….his blood pictures were mcv>90,all peripheral counts decreased and hypersegmented 53)a 5 year boy presented with……with subconjunctival neutrophils…what is your diagnosis?? a: acute leucopenia b:aml haemorrhage.what is your diagnosis??ans:whooping cough c:megaloblastic anemia 54)a 37 yr female presented with h/o vertigo.no tinnitus.no hearing 31)a patient presented with h/o gum bleeding for last 2 days ,there is loss….she has latency of 15 seconds….what is her no cyanosis normal skin pinch.he had joint pain 1 day back.which diagnosis??ans:bppV(benign paroxysmal positional vertigo) would be true regarding this?? a: aptt decreased b:pt decreased c:low 55)a 10 yr old child present with h/o impaired hearing since 3-4 days.o/e there is retracted tympanic membrane and conductive 32)regarding ab+ve blood group which is true??a: no agglutinin b:no hearing loss.what is the diagnosis??a: csom b:asom c: ome 33)which structure is supplied by obturator nerve???a: sartorius b: 56)a 50 yr old patient presented with h/o nasal bleeding….his adductor magnus c:hamstring vitals……what would u do??ans:anterior nasal packing 34)while doing cystic artery ligation in cholecystectomy which 57)one question from seborrhoic dermatitis ligament has to be cut?? a: falciform ligament b:hepatoduodenal c:gastroduodenal d:gastrocolic 58)follwing would be diagnosis of hiv??a:cd4 count<300/mm3 b: 200/mm3 c:400/mm3 d:500/mm3 35)a 49 yr old female presented with h/o acute retention of urine.on examination…there is firm and cysticmass protruding toward pouch 59)one question was from heat stroke( an army man with h/o heavy of douglas.what would you do?? a:transurethral catheterization b:usg exercise……temp>104 degree f .without sweating….what is your c:suprapubic catheterization 36)a 10 yr old boy presented with h/o ballooning of prepuce and acute 60)one question was from waterhouse friedrichson syndrome( a 15 yr retention of urine.what is the diagnosis??a: posterior urethral valve boy with h/o fever with vomiting and rashes on ….his bp 70/50.what b:pinhole meatus is your diagnosis??) 37)in mitral stenosis which is most unlikely??a: left atrial dilatation b: 61)one question was from conn's adenoma…pt's history given.with lvh c:pulmonary htn d:right ventricular hypertrophy htn …serum sodium was high.and k+ low. 38)mid diastolic murmur heard in??a: ms b:mr c:tr 62)a 2 year child with h/o chest infection.failure to thrive.o/e rr>50/min,no cyanosis……what is your diagnosis???ans:vsd 39)you are posted in phc…a 50yr pt. presented with h/o chest pain….sob….on ecg there is st elevation in lead v2-v5 , avl.what will 63)a pt.with h/o…….on x-ray examination boot shaped heart was you not immediately ???a:refer to tertiary centre b:administer high seen.what is your diagnosis???ans:tetralogy of fallot dose aspirin c:02 2litre/min d:give morphine 64)a patient presented with h/o winging of scapula…damage to which 40)how does buscopan(hyoscine butyl bromide) act??a:m2 receptor structure would have cause this???a:axillary nerve b:radial nerve antagonist b:m1 receptor antagonist c:m3 receptor antagonist d:m c:long thoracic nerve receptor agonist 65)a pt. presented with h/o fall on outstretched hand….with shoulder 41)cause of jaundice in pregnancy ?? ans:acute fatty liver dislocated anterior.which is the nerve damage???a: brachial b:radial nerve c:median d:axillary nerve 42)a tourist from england went to thamel ….took fried rice.vanilla sausage…after 4 hours he developed vomiting…diarrhea….which 66)a patient was transfuse 6 units of whole blood…which is false organism is responsible for this???a:staph aureus b:clostridium regarding this??a:thrombocytopenia b ic c:hypocalcemia perfringes c:e. toxigenic e.coli d:bacillus cereus 43) regarding rheumatic heart disease what is the sequence of valve 67)a patient known case of hiv presented with h/o white plaques on oral cavity.on staining pseudohyphae was seen.which organism would have cause this??a:histoplasmosis b:cryptococcus c:candida 44)which poisoning result in optic atrophy ?? ans:methyl poisoning albicans d:blastomycosis 45)a 45 yr female presented with flat topped pinkish lesion ….on 68)a 55 year pt. presented with h/o dvt.which investigation would volar aspect of hand??what is ur diagnosis.??ans:lichen planus you like to send?? a:usg b: venograph c: ct d:doppler 46)in macconkey's agar medium which causes lactose 69)a patient present with h/o suggestive of abdominal aortic fermentation?ans :enterobacter aneurysm.which test would you like to send for diagnosis?? a:ct abdomen b:usg abdomen c:aortogram 47)one question was from throttling.idon't remember the question 70)there was one question from epidemics 48)cornea is supplied by which nerve?a:maxillary others were b: 71)after polio which disease is targeted for eradication from 12)one case was given regarding asthma….first fluticasone was nepal??ans:measles(not sure.check out yourself) given.then changed to budesonide.then beclomethasone.what is true regarding this??a:beclomethasone is more potent b: budesonide is 72)which drug combination would likely responsible for congestive more potent c:fluticasone is more potent c:all r equal in potentcy cardiac failure??a:atenolol+nifedipine b:atenolol+verapamil c:atenolol+diltiazem c:atenolol+amlodipine 13) which decreases gastric secretion>??ans:somatostatin 73)a 32 yr patient with a h/o neck swelling(thyroid) since 20 14) a 40 year old patient …with h/o …had fracture femur…what is years….for the last one month the mass has increased rapidly. the volume of blood loss??a:1000 ml b:1500 ml c:2000 ml d: 2500 ml 32 o/emass is hard ……what is most likely diagnosis??a:anaplastic ca b:papillary ca thyroid c: follicular ca 15)a 45 year pt with h/o diabetes…presented with multiple ulcer in the nape of the neck.what is your diagnosis??ans:carbuncle 74)by direct ophthalmoscope image is magnified how many times ??a:10 times b:20 times c:5 times d:15 times 16) a pt. presented with h/o 14 hours of wound……what would you do??ans: debridement cleaning and antibiotics 75)a patient with h/o discomfort on eye since 2 days…o/e dendritic ulcer was seen.what is the diagnosis?? a: herpes simlex infection 17) kancha 14 year old boy with loss of consciousness , uprolling of b:fungal ulcer c:bacterial infection eye….had his wet pant….what is your diagnosis?? a:anxiety neurosis 76)an obese patient with recent loss of weight….his random blood glucose level is 250 mg%.what would you like to do?? a: advice 18)regarding arf which is not the major criteria??ans: leucocytosis exercise b: start glitazone c: start insulin d:start metformin 19)match the best?? there were two right answer a:x-linked recesive: 77)a 2 yrchild presented with h/o bleeding per rectum……o/e maroon haemophilia another was x linked dominant: vitamin d resistant colored stoll was seen….tenderness on right iliac fossa??what is yourdiagnosis??a:acute appendicitis b:acute intussuception c:meckel's 20)a 55 years male presented with loc since 7 hours …he had h/o alcohol intake history for 20 years.what would be your first line of 78)one question was from juvenile rheumatoid arthritis management??a:start 25% dextrose immediately b:inj thiamine c:wait for blood glucose level 79)one was from rheumatoid arthritis 21)all are true regarding parasympathetic activity except ??ans : 80)one question was from breast milk re: nmcle chaitra 2065 questions - 10-04-2009, 12:33 am 22) true regarding parasympathetic action in eye?? ans: ciliary muscle will update other questions soon. 1)in all condition dpt vaccine is given except??ans 23) a patient with h/o trauma by while playing foot ball on epigastrc region 2 weeks back.on examination fluctuant cystic mass was found at epigastrium.what is the diagnosis??a seudopancreatic cyst 2) a 8 kg child with sunken eyes….eager to drink….what would you b:rectus sheath hematoma c:false aneurysm of aorta do??ans:600ml fluid in first 4 hours 3)which diseas transmit both by droplet and food borne??a:cholera b clinical vignettes (topics only .total 10*5=50) 1.organo phosphorous poisoning 4)cap most common organism???ans:s. pneumoniae 2.extradural hematoma 5) a pt. came with h/o……he couldn't see on left temporal region and 4.t.b meningitis 5.cns infection rt. nasal region……where is the lesion??ans 7.active phase of labour ? aph 6)tribadism is a condition characterized by??a:sex of man with 8.acute rheumatic fever woman b: sex of woman with woman c:sex of man with man d:sex of 9.seizure disorder 7)one question was related to amniotic fluid.i couldn't remember options.they were regarding electrolytes value of amniotic fluid 8) a 15 yr old student….with pain on medial epicondyle while writing….what is your diagnosis?? a:tennis elbow b:golfer's elbow c:student elbow 9) a 6 yr old child with u/l nasal obstruction….foul smelling discharge.what is the diagnosis?? ans: fb in nose 10)all are true regarding dentition except??ans: 3rd molar in 12 years 11)one case was from open angle glaucoma….age group related to this??a:<40 years b: 20-40 years c:>8 years d:>40 years

Source: http://www.nmss.org.np/NMCLE.pdf

addictionsuisse.ch

Dopage au quotidien Un document de base d'Addiction Suisse 1. Introduction A en croire les médias, on assiste à une augmentation de la tendance à recourir aux artifices les plus divers pour augmenter sa performance au travail et dans la vie privée. Même des personnes en parfaite santé n'hésitent pas à intervenir dans le fonctionnement de leur corps. Une pratique qui n'a rien de nouveau. Dans le domaine du physique ce souci d'optimisation, nourri par l'obsession contemporaine de la beauté et de la performance, se traduit déjà, du moins dans certains milieux, par la chirurgie esthétique, la médecine sportive et les médicaments « lifestyle » (par exemple les produits anti-âge). En matière de sport, le dopage a déjà provoqué un vaste débat éthique et philosophique, non seulement en raison des risques qu'il comporte pour la santé, mais surtout du manque de fair-play dont les sportifs de compétition font preuve, de l'avis général, en se procurant ainsi des avantages injustifiés sur leurs adversaires. Cette controverse a débouché sur l'établissement de règles concernant l'utilisation des produits dopants. En revanche, la discussion sur l'emploi de ces derniers dans le cadre de la vie privée et professionnel e ne fait que commencer.

cesp-2016.vjf.inserm.fr

This article appeared in a journal published by Elsevier. The attached copy is furnished to the author for internal non-commercial research and education use, including for instruction at the authors institution and sharing with colleagues. Other uses, including reproduction and distribution, or selling or licensing copies, or posting to personal, institutional or third party